2
$\begingroup$

$\lim_{x\to1}\frac{x + \sqrt{x}}{\sqrt{x-1}}$ $\lim_{x\to1}\frac{x - \sqrt{x}}{\sqrt{x-1}}$

Lately I've been trying to satisfy some curiosity about the nature of limits and I found this example, it's really bugging me that I can't solve it. It seems so simple, yet when I attempt to solve it, it's like I'm trapped in a loop. Everything I do, undos the previous action.

I'll take anything you can give, a solution, a good hint... I tried manipulating the expression without changing it, but I can't get through it.

$\endgroup$
4
  • 4
    $\begingroup$ The numerator goes to $2$ while the denominator goes to $0$. The limit is infinity. Did you mean $x\color{Red}-\sqrt{x}$, in which case the limit goes to $0$? $\endgroup$
    – anon
    Mar 9, 2012 at 10:21
  • $\begingroup$ For x=(+1), it's undefined. You may try to take the square of the equation I think... $\endgroup$ Mar 9, 2012 at 10:22
  • $\begingroup$ Wait, that's the conclusion? That simple? I thought that was it, I just kept pushing and pushing to get something explicit. Nifty. No, thanks, now it's all clear. :D $\endgroup$ Mar 9, 2012 at 10:24
  • $\begingroup$ How would it go if it were $x - \sqrt{x}$? $\endgroup$ Mar 9, 2012 at 10:29

2 Answers 2

3
$\begingroup$

The numerator tends toward $2$ while the denominator tends toward $0$; the form $2/0$ is not indeterminate at all, so no energy needs be spent beating it into shape when we can see for a fact just from this that the limit is $\infty$ (i.e. for any $N>0$ as large as you want there is a neighborhood of the argument $x=1$ for which the ratio is larger than $N$ throughout the entire neighborhood).

In the alternate situation, with a minus sign we can multiply/divide by the numerator's conjugate,

$$\frac{\sqrt{x}(\sqrt{x}-1)}{\sqrt{x-1}}\cdot \frac{\sqrt{x}+1}{\sqrt{x}+1}=\frac{\sqrt{x}\sqrt{x-1}}{\sqrt{x}+1}.$$

This allows for us to directly plug in $x=1$.

$\endgroup$
0
-2
$\begingroup$

1+√1=2; however √1−1=0

2/0 -> infinity This simple...

$\endgroup$

You must log in to answer this question.

Not the answer you're looking for? Browse other questions tagged .